LSAT and Law School Admissions Forum

Get expert LSAT preparation and law school admissions advice from PowerScore Test Preparation.

 Administrator
PowerScore Staff
  • PowerScore Staff
  • Posts: 8929
  • Joined: Feb 02, 2011
|
#91285
Complete Question Explanation

The correct answer choice is (A).

Answer choice (A): This is the correct answer choice.

Answer choice (B):

Answer choice (C):

Answer choice (D):

Answer choice (E):

This explanation is still in progress. Please post any questions below!
User avatar
 rdee81
  • Posts: 18
  • Joined: Aug 23, 2021
|
#95051
i got this answer right, but i was unsure of myself. I was down to the last 3 answers. A C or E. Answers C and E were from the same rule. I was successful in crossing out 2 wrong answers. In a situation like this how do i come to the right answer?
 Rachael Wilkenfeld
PowerScore Staff
  • PowerScore Staff
  • Posts: 1392
  • Joined: Dec 15, 2011
|
#95083
Hi rdee,

Crossing out wrong answers can be a great strategy. I like doing as much up front as I can. So for me, I made a local diagram, putting H in 4. Then visually I looked at what part of my rules were most likely to be an issue. The H1 conditional won't be relevant, because H isn't in 1. The sequence of L-K-F is relevant, as is the MJ block. I'm not yet sure how the F6 conditional would come into play.

Initially, I focused on the MJ block because it seems most limited. The block could either go 1 and 2, 2 and 3, or 5 and 6. I looked at each option one by one to see if it indicated something about this local condition.

If MJ are 1 and 2, that means L is 3, K is 5, and F is 6. Oh! F in 6 triggers a conditional that requires J in 3. But J is in 2 here. So MJ cannot be 1 and 2.

If MJ are 2 and 3, L would be 1, K would be 5, and F would be 6. That also triggers the F in 6 conditional, but here, J is in 3, so it's ok. This is a possible solution. I don't want to stop here though. I want to make sure there are no other solutions. This one doesn't let me know if the answer is answer choice (A) answer choice (C) or answer choice (E).

If MJ are 5 and 6, we have L in 1, K in 2, and F in 3. There! In both possible solutions, L is in 1. That's our must be true answer.

Hope that helps!

Get the most out of your LSAT Prep Plus subscription.

Analyze and track your performance with our Testing and Analytics Package.